Difference between revisions of "2005 AMC 10B Problems/Problem 5"

(Problem)
(Redirected page to 2005 AMC 12B Problems/Problem 3)
 
(2 intermediate revisions by 2 users not shown)
Line 1: Line 1:
{{empty}}
+
#REDIRECT[[2005 AMC 12B Problems/Problem 3]]
== Problem ==
 
Brianna is using part of the money she earned on her weekend job to buy several equally priced CDs. She used one fifth of the money to buy one third of the CD's. What fraction of the money will she have left once she buys the CD's?
 
 
 
<math>\mathrm{(A)}\ {{{\frac{1} {5}}}} \qquad \mathrm{(B)}\ {{{\frac{1} {3}}}} \qquad \mathrm{(C)}\ {{{\frac{2} {5}}}} \qquad \mathrm{(D)}\ {{{\frac{2} {3}}} \qquad \mathrm{(E)}\ {{{\frac{4} {5}}}}</math>
 
 
 
== Solution ==
 
{{solution}}
 
== See Also ==
 
*[[2005 AMC 10B Problems/Problem 4 | Previous problem]]
 
*[[2005 AMC 10B Problems/Problem 6 | Next problem]]
 
*[[2005 AMC 10B Problems]]
 

Latest revision as of 17:27, 28 June 2011